Difference between revisions of "1971 IMO Problems/Problem 3"

(Created page with "Prove that the set of integers of the form 2^k - 3(k = 2; 3; ...) contains an infinite subset in which every two members are relatively prime.")
(No difference)

Revision as of 12:40, 20 October 2013

Prove that the set of integers of the form 2^k - 3(k = 2; 3; ...) contains an infinite subset in which every two members are relatively prime.